nième nombre premier

bonjour,

en admettant le TNP, comment prouver que le nième nombre premier pn est équivalent en l'infini à n * log n (log désignant le logarithme népérien) ?
je sais qu'une preuve est assez simple et figure par exemple dans un livre de K. Madère, mais pourtant je ne sais plus du tout comment faire.
par avance merci de me proposer des pistes.
gaetan michel
«1

Réponses

  • J'appelle $p_n$ le $n$-ième nombre premier.

    On sait que, pour tout $n$, on a $\pi(p_n) = n$. Or, comme $p_n$ tend vers $+\infty$, ça donne, d'après le TNP : $\dfrac{p_n}{\ln(p_n)} \sim n$, ce que j'appellerai l'équation $(1)$.

    Comme les deux membres tendent vers $+\infty$, on a le droit de prendre le $\ln$ dans les équivalents : $\ln(p_n)-\ln(\ln(p_n)) \sim \ln(n)$, i.e $\ln(p_n) \sim \ln(n)$ car $\ln(\ln(p_n))$ est négligeable devant $\ln(p_n)$ en $+\infty$.
    En combinant cela avec l'équation $(1)$, on a donc : $\dfrac{p_n}{\ln(n)} \sim \dfrac{p_n}{\ln(p_n)} \sim n$. D'où $p_n \sim n\ln(n)$.

    Et réciproquement, on peut retrouver le TNP à partir de ce dernier équivalent.
  • bonjour et merci beaucoup guego pour cette réponse rapide.
    gaetan michel
  • Salut, ça m'intéresse aussi cette démonstration, question : ça veut dire quoi : $ \pi(p_n) = n$ ?
  • Au hasard : le $n$-ième nombre premier est $p_n$ ?
    Algebraic symbols are used when you do not know what you are talking about.
            -- Schnoebelen, Philippe
  • Non mais c'est quoi cette application $\pi$ ?
  • Le rang d’un nombre premier ?
    Algebraic symbols are used when you do not know what you are talking about.
            -- Schnoebelen, Philippe
  • Euh ... Désolé d'enchainer encore sur une question, mais, c'est quoi le rang d'un nombre premier ?
  • nombre premier : 2 3 5 7 11 ...
    rang : 1er 2ème 3ème 4ème 5ème ...
  • Ah ok. Merci.
  • A ce propos, qui connaitrait un pdf vers la plus simplissime de chez simplissime preuve du TNP, sans trop de prérequis??
    Aide les autres comme toi-même car ils sont toi, ils sont vraiment toi
  • Pour Jona : $\pi(x)$ est le nombre de nombres premiers inférieurs ou égaux à $x$. Par exemple $\pi(10) = 4$.

    CC : Ce pdf est assez pédagogique et détaillé, je trouve : http://www.math.gatech.edu/\~{}mbaker/pdf/pnt2.pdf

    [Activation du lien. :) AD]
  • D'accord.

    PS : ton lien ne marche pas : erreur 404.
    [Recommence. ;) AD]
  • Merci Guego,

    on y trouve un joli lemme court où le $\vartheta (x)$ est la somme des ln(p) pour pour premier $\leq x$

    j'ai "photographié" le passage je le poste:
    12261
    Aide les autres comme toi-même car ils sont toi, ils sont vraiment toi
  • Ces démonstrations sont tout à fait classiques et se trouvent dans {\bf tous} les ouvrages de TAN. $\theta(x) := \sum_{p \leqslant x} \log p$ désigne la première fonction de Tchebichef.

    Borde.
  • ..et $\vartheta$ n'est pas un $v$ !
  • Je reposte la preuve la plus courte (je crois) de D.J Newman:

    http://mathdl.maa.org/images/upload_library/22/Chauvenet/Zagier.pdf
  • Cette preuve est effectivement plus courte, et est également reprise dans de nombreux ouvrages, notamment des livres récents.

    Elle reste néanmoins non élémentaire, au sens (habituel) où la variable complexe n'est pas éliminée, et, surtout, elle ne donne aucune estimation de l'erreur, comme le montre le document fourni par Olib. Elle est donc surtout utilisée à des fins pédagogiques à visées non professionnelles.

    Une autre démonstration plus courte existe, assez peu connue, et pourtant elle témoigne d'une très grande ingéniosité de la part des auteurs de cette preuve. On peut la trouver dans le récent ouvrage suivant :

    {\bf Iwaniec et Kowalski}, {\it Analytic number theory}, AMS {\bf 53}, 2004, 40--42.

    L'un de ses avantages est que l'on reste dans le domaine de l'arithmétique, puisque les auteurs démontrent le TNP version fonction de Möbius. Ceci dit, comme elle travaille également avec les propriétés analytiques de $\zeta$, elle n'est pas considérée comme étant pleinement élémentaire.


    Borde.
  • Merci pour ces liens, et merci Egoroff pour le latex $\vartheta$, j'ai corrigé directement...
    Aide les autres comme toi-même car ils sont toi, ils sont vraiment toi
  • à Borde: je devine que c'est archiclassique, en fait j'ai fait la photo d'écran pour la raison suivante: il a été extrêmement médiatisé auprès du public amateur comme moi tout plein de résultats allant dans le sens qu'il y a "beaucoup" de nombres premiers.

    Il m'apparaissait pas inintéressant de poster cette photo de quelques lignes élémentaires de niveau terminale qui montre qu'il n'y en a pas "tant que ça" (puisqu'en additionnant leur ln...)

    D'ailleurs, peut-on montrer en quelques lignes de niveau terminale qu'il existe un nombre $c>0$ tel que quand $x$ est assez grand, le nombre de nombres premiers entre $0$ et $x$ dépasse $cx/\ln(x)$? (Je regarderai dans le pdf lié plus tard, là je n'ai pas le temps, peut-être que ça y est fait)
    Aide les autres comme toi-même car ils sont toi, ils sont vraiment toi
  • Bonjour,

    Christophe, on peut démontrer l'existence de $c$, et même lui donner une valeur.

    1/ Si $d_n$ est le ppcm des $n$ premiers entiers, alors $\pi(n)\geqslant\ln(d_n)/\ln(n)$. En effet, si $d_n=\prod p^{\alpha_p}$, alors $\alpha_p\leqslant\ln(n)/\ln(p)$, donc $\ln(d_n)\leqslant \sum_{p\le n}\ln n = \pi(n)\ln n$ (car tous les facteurs premiers de $d_n$ sont $\leqslant n$.

    2/ On a $d_{2n+1}\geqslant 4^n$. Pour ça on peut considérer $I_n=\int_0^1x^n(1-x)^n{\rm d}x$. Vu que $x(1-x)\leqslant 1/4$ pour $x\in[0,1]$, on a $I_n\leqslant 1/4^n$. D'autre part,
    \[I_n=\sum_{k=0}^n\frac{(-1)^k\binom{n}{k}}{n+k+1}.\]
    Mais $d_{2n+1}$ est multiple de $n+k+1$ pour $k\in[0,n]$, donc $d_nI_n\in\Z$. Comme $d_nI_n>0$, on a $d_nI_n\geqslant1$, d'où la majoration recherchée.

    3/ Vu que $d_{2n+2}=d_{2n+1}$ et vue la minoration précédente, $\ln(d_n)\geqslant\left[\frac{n-1}2]\ln 4\geqslant (n+1)\ln 2$, d'où
    \[\pi(n) \geqslant \frac{n+1}{\ln n}\ln 2.\]

    Si quelqu'un a une idée de l'endroit d'où sort ce truc, je suis intéressé. Je l'avais noté dans un coin, mais je n'ai pas noté la source :-(.
  • A c'est gigantesque: 2 preuves accessibles en terminale établissant que:

    il existe des nombres positifs réels $0<a<b$ et un nombre $M$ tel que pour $x\in \R$ avec $x>M$

    le nombre de nombres premiers compris entre 0 et $x$ est compris entre $a \frac{x}{ln(x)}$ et $b \frac{x}{ln(x)}$

    A priori à l'heure actuelle, ne sont pas connues de preuves simples du TNP...

    Merci pour ces preuves!!!
    Aide les autres comme toi-même car ils sont toi, ils sont vraiment toi
  • Merci qg. Il va falloir que je me décide un jour à l'acheter, ce livre d'O. Bordellès ! :)
  • N' y aurait-il pas un argument simple et court, pas une preuve valable, mais simplement une "idée" vague qui le rendrait le TNP intuitif???

    Je pense à un argument, par exemple, où on se fiche que les séries formelles ne convergent pas, etc... où on se contente d'identifications formelles symboliques??
    Aide les autres comme toi-même car ils sont toi, ils sont vraiment toi
  • On peut peut-être y arriver avec des arguments combinatoires et le théorème fondamental de l'arithmétique.
  • {\it Il va falloir que je me décide un jour à l'acheter, ce livre d'O. Bordellès}

    C'est une bonne idée, ça ! :)-D


    Borde.
  • EDIT: je précise que la suite est un délire inspirant et n'a aucune valeur AVANT lol

    Le pdf, offre un calcul dont ressort une idée: si on note $s(p)$, la somme des $1/p^n$, symboliquement $s(p)=1/(1-p)$, le produit des inverses des $s(1/p)$ quand p parcourt les premiers représentent la proportion symbolique des suites $u$ telle que pour tout entier $n$, $u(n)$ n'est pas un multiple de $p$.

    En développant le produit, on obtient l'inverse de la somme des inverses de tous les entiers que je note $1/ln(Z)$ où $Z$ ne désigne rien du tout à part un symbole intuitif. Pourquoi "ln"? Parce qu'en développant le produit, on tombe effectivement (toujours symboliquement) sur un dénominateur égal à la somme des inverses des entiers... chacun étant compté une et une seule fois

    Le théorème chinois inspire l'idée que "tirer" un [size=x-small](ultrafiltre d')[/size]entier au hasard aboutit à ce qu'il ait une chance sur $ln(Z)$ de n'être divisible par aucun nombre premier "standard"...

    Si on veut bien "croire" qu'il correspond à un nombre premier "virtuel" dans ce cas, et pas dans les autres, effectivement, ça correspond à une proportion de $Z/ln(Z)$ nombres premiers lol :)-D

    Toutes mes excuses aux pros de "GDNiser" une spécialité qu'ils aiment tant, je cherchais une "image" mentale.

    Je suis effectivement convaincu que le TNP est vrai sans connaitre de preuves... Niveau d'incertitude à déterminer avec mon psy... S'il n'y avait pas le théorème choinois où j'ai franchement l'impression de tricher (ie chinois$\to$ indépendance probabiliste hum hum) le reste se formaliserait gentiment avec des jonglage d'ultrafiltres et Ramsey
    Aide les autres comme toi-même car ils sont toi, ils sont vraiment toi
  • Merci, en fait je crois que ce théorème illustre à merveille que les problèmes de convergence peuvent être très difficiles. Finalement, ce TNP, ce n'est pas tant d'y croire qui est difficile (on jette tout scrupule à la poubelle, et on dit que "tout converge"), c'est de le prouver avec soin (ie boucher les trous où on a supposé que tout converge).

    Je ne suis pas assez cultivé, mais je pense qu'il peut y avoir des preuves alternatives en mettant des mesures adéquates sur l'ensemble des ultrafiltres sur $\N$, autrement dit de remplacer des pour tout de niveau arithmético-analytiques par des "il existe au niveau de $P(P(\N))$", procédé d'ailleurs qui est courant et qui fait découvrir parfois des paradoxes quand il y a obstruction (exemple Banach-Tarski)

    D'ailleurs une petite conjecture inspirée de ces nonobstructions éventuelles:

    Dans un anneau quelconque, il existe une application injective $f$ de l'ensemble des idéaux premiers dans l'ensemble des idéaux, décroissante (donc strictement) pour l'inclusion.

    Je ne suis pas sûr qu'elle soit vraie, mais ce phénomène d'inversion serait, s'il était connu, un beau joyau...
    Aide les autres comme toi-même car ils sont toi, ils sont vraiment toi
  • Voici par contre un axiome qui peut être utilisé à volonté, et qui serait peut-être utile dans ces sujets d'arithmétique. Habituellement, ces axiomes servent à autre chose et il n'y a pas de communication, c'est dommage. Attention, il est incompatble avec l'axiome du choix, mais accepté et sécurisé par les grands cardinaux

    Soit $T$ l'ensemble des parties finies de $\N$ et $U$ l'ensemble des suites d'éléments d'un ensemble fini $F$.

    Soit $f$ une application de $T$ dans $U$.

    Alors il existe une suite $S$ de couples $(v_n;A_n)\in U\times T$ et un entier $p$ telle que:

    Les ensembles $A_n$ partitionnent l'ensemble des entiers $>p$

    Pour tout entier $n$ et toute partie $B\subseteq A_n$, pour tout entier $q<min(B)$:

    $f(B)(q)=v_n(q)$


    En général, en choisissant bien $f$, la $S$ obtenue rigidifie plein de phénomènes de convergence de manière naturelle et "facile" à établir. C'est une généralisation du Ramsey prouvable.

    On peut par exemple à essayer avec $f$ qui à $A$ associe la fonction caractéristique de l'ensemble des premiers qui divisent au moins un élément de $A$.
    Aide les autres comme toi-même car ils sont toi, ils sont vraiment toi
  • Bonjour,

    CC: "S'il existe des nombres positifs réels $0<a<b$ et un nombre $M$ tel que pour $x\in \R$ avec $x>M$

    le nombre de nombres premiers compris entre 0 et $x$ est compris entre $a \frac{x}{ln(x)}$ et $b \frac{x}{ln(x)}$ ".

    au hasard de mes pérégrinations arithmétiques:
    -> concours ENAC 1989: $a= Ln(2)$ et $b=2Ln(2)$
    -> Thèmes d'Arithmétique: $a=\frac{1}{2}$ et $b=\frac{5}{2}$

    Amicalement.

    [Edit: pourtant, l'aperçu était lisible...]
  • Exact, Bernard, je n'ai pas cherché à obtenir les meilleures constantes possibles, car il faut le recours à l'analyse complexe et à des estimations fines du nombre de zéros non triviaux de $\zeta$.

    En revanche, je suis assez surpris par ton $b = \log 4$ sachant que la meilleure constante valide pour tout $x > 0$ est de $1,25506$, obtenue en 1962 par Rosser \& Schoenfeld en utilisant les méthodes que je viens d'indiquer. Ainsi, plusieurs scénarii sont possibles pour cette valeur de $\log 4$ :

    (i) Ou bien il y a un terme d'erreur, du genre :

    $$\pi(x) \leqslant \frac{x \log 4}{\log x} + O \left ( \log x \right ).$$

    (ii) Ou bien l'estimation est vraie pour des valeurs de $x$ assez grandes.

    Peux-tu nous en dire plus ?

    Merci,


    Pour BadWolf,

    Je poursuis le message de qg77.

    La méthode que tu cites a été introduite par Mohan Nair en 1982 et peut être généralisée de la manière suivante, sachant que tout repose sur l'égalité $d_n = e^{\Psi(n)}$ avec $d_n = \mathrm{ppcm}(1,\dotsc,n)$ et $\Psi(x) = \sum_{n \leqslant x} \Lambda(n)$ est la seconde fonction de Tchebichef :

    (i) Si $P_n \in \mathbb{Z}[X]$ est un polynôme de degré $\leqslant n - 1$, alors on a :

    $$d_n \int_{0}^{1} P_n(x) \mathrm{d}x \in \mathbb{Z}.$$

    (ii) Si $\int_{0}^{1} P_n(x) \mathrm{d}x > 0$, alors on en déduit que :

    $$\Psi(n) \geqslant \log \left ( \frac{1}{\int_{0}^{1} P_n(x) \mathrm{d}x} \right ).$$

    Nair a choisi un polynôme de la famille des $P(x) = x^k(1-x)^k$ ce qui entraîne une estimation du type $\displaystyle{\liminf_{x \rightarrow \infty} \frac{\Psi(x)}{x} \geqslant \log 2}$. On peut obtenir un peu mieux par d'autres choix de polynômes, mais il est connu que tout polynôme en une variable positif ou nul sur $[0,1]$ ne permet pas d'atteindre la minoration $\displaystyle{\liminf_{x \rightarrow \infty} \frac{\Psi(x)}{x} > 0,9}$.

    {\bf Référence}. {\bf H.G. Diamond}, {\it Elementary methods in the study of the distribution of prime numbers}, Bull. Amer. Math. Soc. {\bf 7} (1982), 553--589.


    Borde.
  • Bonjour Olivier,

    [Edit: $Log$= Log néperien (notation de l'énoncé) ==> $Log(4)=1.3862$ > 1.25506 ]

    Ce problème figure dans le même recueil que celui dans lequel je suis allé pêcher la QDLn°30 , deux problèmes que j'avais faits il y a quelques années. Alors, bien entendu, la question exacte est: ouvrez les " "

    En déduire que pour tout réel $\epsilon$ , strictement positif, il existe un entier $N$, tel que pour tout entier $n$ supérieur à $N$ , on ait:
    $$Log(2) - \epsilon < \frac{\pi(n) Log(n)}{n} < 2Log(2) + \epsilon$$

    Le record de Rosser & Schoenfeld tient toujours.

    Amicalement.
  • OK, c'est bien ce que je pensais, il s'agit d'une estimation asymptotique (assez faible, d'ailleurs, car on ne donne pas la dépendance en $\varepsilon$ de l'entier $N=N(\varepsilon)$, et on ne fournit pas non plus la valeur des $\varepsilon$ en terme de fonctions de $n$). J'ai voulu éviter de mettre de telles estimations dans mon livre, leur préférant des inégalités effectives, quitte à perdre sur les constantes ($5/2$ n'est pas optimale). La méthode de Nair s'est alors imposée naturellement.

    {\it Le record de Rosser & Schoenfeld tient toujours}

    Il sera difficile de faire mieux.


    Borde.
  • bonjour
    j'ai une petite question:
    pourquoi cherche t'on une constante pour améliorer l'estimation du
    $Ln.x$ alors qu'il faut enlever 1 plus une variable Inconnue I dont la différence d entre ces $Ln.x$ varie de 0,05....+ ou - lorsque X tend vers l'infini, c'est à dire que la différence d entre ces variables I oscille: d: 0.03... 0.04.... 0.02.... 0.03...etc donc d ne peut être une constante, et comme vous le dite effectivement, on ne pourra jamais être équivalent à : X / pi.X. pour connaître le nombre de premiers
  • Leg,

    Je ne suis pas sûr de comprendre ta question. Je vais tenter une réponse, mais peut-être ne sera-t-elle pas satisfaisante pour toi :
    \begin{enumerate}
    \item D'un point de vue historique, Tchebichef avait déjà obtenu, au milieu du XIXème siècle, deux constantes $0 < c_1 \leqslant 1 \leqslant c_2$ et un nombre réel $x_0$ tels que :
    $$\frac{c_1 x}{\log x} \leqslant \pi(x) \leqslant \frac{c_2 x}{\log x}$$
    pour tout réel $x \geqslant x_0$. Il a paru donc naturel aux chercheurs qui lui ont succédé de voir si l'on pouvait améliorer $c_1,c_2$ et/ou $x_0$, voire d'arriver à un résultat optimal en terme de ces constantes.

    \item On s'est aperçu que ce problème, en apparence anodin, était hautement non trivial. Les chercheurs US Rosser \& Schoenfeld ont finalement abouti (en 1962) à des résultats presqu'optimaux avec l'aide :

    (i) de bons ordinateurs,

    (ii) de la connaissance d'un nombre de plus en plus grand de zéros non triviaux de $\zeta$, d'où l'on déduit que ce problème n'est pas élémentaire,

    (iii) d'une version effective de la célèbre formule explicite de Von Mangoldt.

    \item Les résultats précédents ont par la suite été améliorés puis généralisés à la fonction $\pi(x;a,q)$ notamment par McCurley, Ramaré et Dusart. D'autres spécialistes les ont utilisés pour obtenir des ordres de grandeur explicites d'autres fonctions de nombre premier, dont $p_n$.
    \end{enumerate}
    Cela répond-il à ta question ?

    Borde.
  • Bonjour

    Borde,
    effectivement ta réponse va dans le même sens , c'est moi qui n'avait pas fait attention au fait que $\pi(x)$ était compris entre ces deux valeurs

    $$\frac{c_1 x}{\log x} \leqslant \pi(x) \leqslant \frac{c_2 x}{\log x}$$

    Et merci pour l'explication.
    leg.
  • OK.

    Rappelons que Titchmarsh a proposé une notation pour ce type d'encadrement, très souvent utilisée :

    $$\pi(x) \asymp \frac{x}{\log x}$$

    valide pour $x \geqslant x_0$, car les constantes impliquées sont rarement indispensables en pratique. Attention toutefois à ne pas confondre cette notation avec :

    $$\pi(x) \sim \frac{x}{\log x}$$

    pour $x \rightarrow \infty$, car cela reviendrait à dire que Tchebichef avait démontré le TNP, ce qu'il n'a pas réussi à faire, et ce même si conceptuellement il n'en était pas très loin !


    Borde.
  • je pense que lorsque la route est tracée (Tchebichef ) ...cela devient plus facile...;)

    je trouve d’ailleurs dommage que toutes les personnes ayant participé à l’élaboration d’un théorème ne soit pas cité dans la démonstration finale
  • Pendant qu'on est dans les notations et que Borde est dans le coin, l'implication

    $$\forall f,g \ \ f(x) \sim g(x)\Longrightarrow f(x) \asymp g(x)$$

    est-elle vraie ?
  • Moralement, $f \sim g$ signifie que $f/g \to 1$ et $f \asymp g$ signifie que $f/g$ reste borné entre deux constantes $0<c<C<+\infty$. Tu devrais pouvoir répondre tout seul !
  • Si $c$ est supérieur à $1$, c'est faux, mais je pensais que ces deux constantes devaient encadrer $1$, d'où ma question.
  • Je ne comprends ; quelle implication es-tu en train d'établir ?
  • Sylvain fait un dessin.
  • Je crois que c'est plutôt un problème de logique ; lorsqu'on veut prouver $f \asymp g$, les constantes $c$ et $C$ ne sont pas données à l'avance, il faut prouver qu'on peut trouver $c,C$ telles que ... mais bien sûr $c,C$ dépendent de $f,g$.
  • Non Rémi, je ne fais pas de dessin, j'écoute de la musique ! (:D

    @egoroff : c'est pourtant simple : si avec tes notations $c=2$ et $C=3$ (par exemple), $f/g$ ne peut pas tendre vers $1$. Donc la réponse à ma question est "non, c'est faux".
  • Qu'est-ce qui est faux ?
  • Sylvain si tu part de la conclusion sans avoir les hypothèse de départ, que compte-tu prouver. Oui egoroff il y a un souci de logique.
    Sylvain bois un café cinq minutes et repenses-y après, tu verras.
  • Mais non il n'y a pas de souci de logique, et j'ai déjà bu un café pour écrire, je ne vais pas en reprendre un surtout que je suis en train de vider une canette de coca. Ce qui est faux c'est l'implication que j'avais écrite en LaTeX, egoroff.
  • Tu as écrit :

    C'est pourtant simple : si avec tes notations $ c=2$ et $ C=3$ (par exemple), $ f/g$ ne peut pas tendre vers $ 1$.

    Là il n'y a pas un souci de logique. On suppose que $f/g$ tend vers un 1 et tu si $ c=2$ et $ C=3$ alors c'est pas possible. Ben peut-être que c'est sûr qu'on va pas tomber sur ces valeurs ?? J'ai l'impression que tu cherches un contre-exemple mais que tu oublies l'hypothèse de départ.

    Tu supposes que tu as $f\sim g$ donc que $f/g\longrightarrow 1$ et tu te demandes juste si tu peux trouver deux constantes pour encadrer $f/g$ en jouant avec des epsilon, ça doit se faire non ?

    [La case LaTeX. :) AD]
    [Merci Alain]
Connectez-vous ou Inscrivez-vous pour répondre.